Horizontal inflection point of a parametric polynomial function

Click For Summary
For x=-1 to be a horizontal inflection point of a parametric polynomial function, the first derivative y' must equal zero, leading to the condition a=2/3b. The second derivative should also be evaluated to confirm the inflection point, which has caused confusion regarding the values of a and b. A correction was suggested in calculating the second derivative, resulting in the equations 3a=2b-2 and 3a=b. This highlights the importance of accurately working through derivative calculations to determine the correct parameters. The discussion emphasizes the need for careful analysis in solving polynomial function conditions.
greg_rack
Gold Member
Messages
361
Reaction score
79
Homework Statement
Given ##y=ax^3+bx^2+2x-3##, find the values of ##a## and ##b## for which the function has an horizontal inflection point at ##x=-1##.
Relevant Equations
none
For ##x=-1## to be an *horizontal* inflection point, the first derivative ##y'## in ##-1## must be zero; and this gives the first condition: ##a=\frac{2}{3}b##.

Now, I believe I should "use" the second derivative to obtain the second condition to solve the two-variables-system, but how?
Since it is an inflection point, shouldn't even the second derivative be zero? But the fact that it's an horizontal one is confusing me.
 
Physics news on Phys.org
greg_rack said:
For ##x=-1## to be an *horizontal* inflection point, the first derivative ##y'## in ##-1## must be zero; and this gives the first condition: ##a=\frac{2}{3}b##.
Check this :wink:
greg_rack said:
Since it is an inflection point, shouldn't even the second derivative be zero?
Your thinking is correct!
 
etotheipi said:
Your thinking is correct!
But then, why does ##y''(-1)=0 \rightarrow b=0##, if the correct result should be ##b=2##?
 
greg_rack said:
But then, why does ##y''(-1)=0 \rightarrow b=0##, if the correct result should be ##b=2##?

I think you made an error in working out the second derivative. Shouldn't it be, ##y'(-1) = 3a(-1)^2 + 2b(-1) + 2 = 3a - 2b + 2 \overset{!}{=} 0 \implies 3a = 2b -2##, and then ##y''(-1) = 6a(-1) + 2b \overset{!}{=} 0 \implies 3a = b##?
 
  • Like
Likes greg_rack
etotheipi said:
I think you made an error in working out the second derivative. Shouldn't it be, ##y'(-1) = 3a(-1)^2 + 2b(-1) + 2 = 3a - 2b + 2 \overset{!}{=} 0 \implies 3a = 2b -2##, and then ##y''(-1) = 6a(-1) + 2b \overset{!}{=} 0 \implies 3a = b##?
Yupp, here's the deal... silly me!
Thanks a lot :)
 
  • Like
Likes etotheipi
greg_rack said:
Yupp, here's the deal... silly me!
Thanks a lot :)

Don't worry, I do stuff like that all the time :smile:
 
  • Haha
Likes greg_rack
Question: A clock's minute hand has length 4 and its hour hand has length 3. What is the distance between the tips at the moment when it is increasing most rapidly?(Putnam Exam Question) Answer: Making assumption that both the hands moves at constant angular velocities, the answer is ## \sqrt{7} .## But don't you think this assumption is somewhat doubtful and wrong?

Similar threads

Replies
7
Views
2K
  • · Replies 4 ·
Replies
4
Views
3K
  • · Replies 6 ·
Replies
6
Views
1K
Replies
3
Views
2K
Replies
11
Views
2K
  • · Replies 19 ·
Replies
19
Views
2K
  • · Replies 7 ·
Replies
7
Views
2K
  • · Replies 5 ·
Replies
5
Views
3K
Replies
7
Views
2K
  • · Replies 18 ·
Replies
18
Views
2K